Podprzestrzenie- dowd

Przestrzenie wektorowe, bazy, liniowa niezależność, macierze.... Formy kwadratowe, twierdzenia o klasyfikacji...
Margaretta
Użytkownik
Użytkownik
Posty: 143
Rejestracja: 9 lip 2004, o 15:24
Płeć: Kobieta
Lokalizacja: Police

Podprzestrzenie- dowd

Post autor: Margaretta »

Niech \(\displaystyle{ L_{1} i L_{2}}\) beda podprzestrzeniami skonczenie wymiarowej przesrzeni wektorowej \(\displaystyle{ V}\). Wykazac, ze:
a) jesli \(\displaystyle{ dim(L_{1} + L_{2})=1+dim(L_{1} \cap L_{2}}\), to suma \(\displaystyle{ L_{1} + L_{2}}\) jest rowna jednej z tych podprzestrzeni, a przeciecie \(\displaystyle{ L_{1} \cap L_{2}}\) drugiej.
b) jesli \(\displaystyle{ dim(L_{1} + L_{2})> dimV}\), to \(\displaystyle{ L_{1} \cap L_{2}}\)jest rozne od {0}.
Awatar użytkownika
Sir George
Użytkownik
Użytkownik
Posty: 1145
Rejestracja: 27 kwie 2006, o 10:19
Płeć: Mężczyzna
Lokalizacja: z Konopii
Podziękował: 4 razy
Pomógł: 203 razy

Podprzestrzenie- dowd

Post autor: Sir George »

Margaretta pisze:... jesli \(\displaystyle{ dim(L_{1} + L_{2})=1+dim(L_{1} \cap L_{2}}\), to ...
a czy przypadkiem nie powinno być
\(\displaystyle{ \mbox{dim}L_1\, +\, \mbox{dim}L_2}\)

Bo jak inaczej zdefiniować \(\displaystyle{ L_1\,+\,L_2}\) niż tak:

\(\displaystyle{ L_1\,+\,L_2\ := \ ft\{v\in V\ : \ v=v_1+v_2,\, v_1\in L_1,\, v_2\in L_2\right\}}\) ...
Margaretta
Użytkownik
Użytkownik
Posty: 143
Rejestracja: 9 lip 2004, o 15:24
Płeć: Kobieta
Lokalizacja: Police

Podprzestrzenie- dowd

Post autor: Margaretta »

w zadaniu, w książce jest tak jak napisałam..=/
Awatar użytkownika
Sir George
Użytkownik
Użytkownik
Posty: 1145
Rejestracja: 27 kwie 2006, o 10:19
Płeć: Mężczyzna
Lokalizacja: z Konopii
Podziękował: 4 razy
Pomógł: 203 razy

Podprzestrzenie- dowd

Post autor: Sir George »

Margaretta pisze:w książce jest tak jak napisałam..
To książkę idzie o kant (***) rozbić...

Jeżeli \(\displaystyle{ L_1, L_2}\) są podprzestrzeniami \(\displaystyle{ V}\), to (z tego co pisałem w pierwszym poście) wynika, że również \(\displaystyle{ L_1+L_2}\) jest podprzestrzenią \(\displaystyle{ V}\). Zatem jej wymiar nie może przekraczać wymiaru całej przestrzeni. Czyli punkt b) jest spełniony automatycznie (założenie \(\displaystyle{ \mbox{dim}(L_1+L_2)\,>\,\mbox{dim}V}\) nigdy nie będzie spełnione...)

Moim zdaniem powinno tam być \(\displaystyle{ \mbox{dim}L_1+\mbox{dim}L_2\,>\,\mbox{dim}V}\)


A co do punktu a), to z warunku \(\displaystyle{ \mbox{dim}(L_1+L_2)\,=\,1+\mbox{dim}(L_1\cap L_2)}\) wynika, że kowymiar \(\displaystyle{ L_1\cap L_2}\) jako podprzestrzeni \(\displaystyle{ L_1+L_2}\) jest jeden.
Innymi słowy dla dowolnego \(\displaystyle{ v\in (L_1+L_2)\setminus(L_1\cap L_2)}\) mamy \(\displaystyle{ \mbox{lin}\big(L_1\cap L_2,v\big)\,=\,L_1+L_2}\), co daje już \(\displaystyle{ \mbox{lin}\big(L_1\cap L_2,v\big)\,=\,L_1}\) lub \(\displaystyle{ \mbox{lin}\big(L_1\cap L_2,v\big)\,=\,L_2}\) (skąd wprost wynika już teza)...
ODPOWIEDZ